Soluzioni Seconda prova scritta Matematica 2007 โ€“ Liceo Scientifico di ordinamento sessione ordinaria

PROBLEMA 1

Si considerino i triangoli la cui base รจ $A B=1 \mathrm{e}$ il cui vertice $C$ varia in modo che l’angolo $C \hat{A} B$ si mantenga doppio dell’angolo $A \hat{B} C$.

  1. Riferito il piano ad un conveniente sistema di coordinate, si determini l’equazione del luogo geometrico $\gamma$ descritto da $C$.
  2. Si rappresenti $\gamma$, tenendo conto, ovviamente, delle prescritte condizioni geometriche.
  3. Si determini l’ampiezza dell’angolo $A \hat{B} C$ che rende massima la somma dei quadrati delle altezze relative ai lati $A C$ e $B C$ e, con l’aiuto di una calcolatrice, se ne dia un valore approssimato in gradi e primi (sessagesimali).
  4. Si provi che se $A \hat{B} C=36^{\circ}$ allora รจ $A C=\frac{\sqrt{5}-1}{2}$

SOLUZIONE DEL PROBLEMA 1

Punto 1.

Consideriamo il punto A coincidente con l’origine degli assi cartesiani e il punto B di coordinate $(1 ; 0)$

Facciamo passare per il punto $A$ una semiretta r.

Costruiamo l’asse a del segmento $A B$. Questo interseca la retta r nel punto D .

Disegniamo la bisettrice b’ dell’angolo formato dalla semiretta r con il semiasse positivo delle $x$.

Intersechiamo b con r ; si ottiene il punto C (figura 1 ).

Punto 2.

Al variare della semiretta r attorno all’origine degli assi, il punto C descrive un luogo geometrico $\gamma$ di cui si vuole trovare l’equazione. Se indichiamo con $2 t$ l’angolo BAC, l’equazione della retta r รจ data da:

$y=(\tan 2 t) x$.

La retta b ha equazione:

$ y=(\tan (\pi-t))(x-1)$

Intersecando queste due rette si ottiene il punto C :

$ \left\{\begin{array}{l} y=(\tan 2 t) x \\ y=(-\tan t)(x-1) \end{array}\right. $

Possiamo dunque scrivere:

$ \left\{\begin{array}{l} y=\frac{2 \tan t}{1-\tan ^2 t} x \\ \tan t=\frac{y}{1-x} \end{array}\right. $

Sostituendo $\tan t=\frac{y}{1-x}$ nella prima equazione, dopo alcuni calcoli, si elimina il parametro $t$ tra le due equazioni e si ottiene che il luogo geometrico $\gamma$ ha per equazione cartesiana:

$ 3 x^2-y^2-4 x+1=0 $

Di questa iperbole si deve considerare solo il ramo “di sinistra”, ossia quello che giace nel semipiano definito da $x \leq \frac{1}{3}$ Si tratta quindi di un’iperbole traslata che si puรฒ scrivere in forma canonica:

$ \frac{\left(x-\frac{2}{3}\right)^2}{\frac{1}{9}}-\frac{y^2}{\frac{1}{3}}=1 $

Ponendo $\left\{\begin{array}{l}X=x-\frac{2}{3} \\ Y=y\end{array}\right.$ (traslazione degli assi)

Si ha:

$ \frac{X^2}{\frac{1}{9}}-\frac{Y^2}{\frac{1}{3}}=1 $

Con centro di simmetria nel punto $O^{\prime}\left(\frac{2}{3} ; 0\right)$ e asintoti le rette di equazioni $y= \pm \sqrt{3}\left(x-\frac{2}{3}\right)$.

Soluzione alternativa dei primi due punti del problema

Si consideri il trapezio isoscele ABEC (figura 3).

In questo trapezio la base CE รจ uguale al lato AC (e quindi anche al lato BE). Infatti รจ facile dimostrare che il triangolo AEC รจ isoscele perchรฉ l’angolo CAE รจ uguale all’angolo CEA (che a sua volta รจ uguale all’angolo EAB).

Essendo M il punto medio di CE , ne consegue che CM รจ sempre la metร  del segmento AC .

Consideriamo ora la retta a (asse del segmento AB ) come direttrice e il punto A come fuoco. Possiamo pensare al luogo geometrico $\gamma$ come luogo dei punti C tali che

$\frac{C A}{C M}=2$

Il luogo $\gamma$ รจ quindi una parte di iperbole di eccentricitร  2 che ha un fuoco F nel punto A e una direttrice coincidente con la retta $a$ avente per equazione $x=\frac{1}{2}$.

Considerando il punto A come origine degli assi e l’asse positivo delle $x$ coincidente con AB , si ottiene:

$\frac{\sqrt{x^2+y^2}}{\frac{1}{2}-x}=2$

Quadrando si ottiene:

$\frac{x^2+y^2}{\frac{1}{4}-x+x^2}=4$

da cui si ottiene l’equazione del luogo:

$3 x^2-y^2-4 x+1=0$

Di questa iperbole si deve considerare solo il ramo “di sinistra”, ossia quello che giace nel semipiano definito da $x \leq \frac{1}{3}$,

Punto 3.

Disegniamo le altezze relative ai lati AC e BC come richiesto.

Ragionando sui triangoli rettangoli ABK e ABH , si ha $\overline{A K}=\sin t$ e $\overline{A H}=\sin 2 t$.

Nel triangolo ABC deve essere $t+2 t<\pi$.

Si ricava pertanto che $0<t<\frac{\pi}{3}$.

La funzione si cui viene chiesto il massimo รจ la seguente:

$s(t)=\sin ^2 t+\sin ^2(2 t)$

Studiando il segno della derivata prima

$
s^{\prime}(t)=2 \sin (4 t)+\sin (2 t)
$

si ottiene che il massimo si ha per $t=\arcsin \sqrt{\frac{5}{8}}$.

Usando la calcolatrice, si ottiene $t=0,9117 \ldots \mathrm{rad}=52^{\circ} 14^{\prime} \ldots$.

Punto 4.

Se l’angolo $\mathrm{ABC}=36^{\circ}$, allora l’angolo in A vale $72^{\circ}$ e l’angolo in C vale $72^{\circ}$.

Quindi il triangolo รจ isoscele.

Questo triangolo isoscele si ritrova nella costruzione geometrica del decagono regolare inscritto in una circonferenza (il lato รจ la sezione aurea del raggio).

Quindi AC รจ la sezione aurea di AB , ossia vale la seguente relazione:

$\overline{A C}=\overline{A B} \frac{\sqrt{5}-1}{2}=\frac{\sqrt{5}-1}{2}$

PROBLEMA 2

Si consideri un cerchio $C$ di raggio $r$.

  1. Tra i triangoli isosceli inscritti in $C$ si trovi quello di area massima.
  2. Si denoti con $S_n$ l’area del poligono regolare di $n$ lati inscritto in $C$. Si dimostri che $S_n=\frac{n}{2} r^2 \operatorname{sen} \frac{2 \pi}{n}$ e si trovi un’analoga espressione per l’area del poligono regolare di $n$ lati circoscritto a $C$.
  3. Si calcoli il limite di $S_n$ per $n \rightarrow \infty$.
  4. Si spieghi in che cosa consista il problema della quadratura del cerchio e se, e in che senso, si tratti di un problema risolubile o meno.

Soluzione del problema 2

Punto 1.

Consideriamo la figura 1.

Indichiamo con x la misura dellโ€™altezza HC del triangolo.

Per i secondo teorema di Euclide, si ha:

$
\begin{aligned}
& \overline{H B}=\sqrt{x(2 r-x)} \
& \operatorname{con} 0 \leq x \leq 2 r
\end{aligned}
$

L’area del triangolo ABC รจ quindi:

$S(x)=x \sqrt{x(2 r-x)}$

Derivando si ottiene:

$S^{\prime}(x)=\frac{x(3 r-2 x)}{\sqrt{x(2 r-x)}}$

L’area รจ massima per $x=\frac{3}{2} r$.

In questo caso il triangolo รจ equilatero.

Punto 2.

Consideriamo un poligono regolare di $n$ lati. Lo possiamo suddividere in $n$ triangoli isosceli uguali. L’area di uno di questi triangoli รจ data da

$$
S_{\text {triangolo }}=\frac{1}{2} r^2 \sin \left(\frac{2 \pi}{n}\right)
$$

L’area del poligono regolare sarร  pertanto:

$$
S_n=\frac{n}{2} r^2 \sin \left(\frac{2 \pi}{n}\right)
$$

Il poligono regolare circoscritto con lo stesso numero di lati avrร  area:

$$
S_n^{\prime}=n r^2 \tan \left(\frac{\pi}{n}\right)
$$

Punto 3.

Il limite richiesto รจ il seguente:

$\lim {n \rightarrow \infty} S_n=\lim {n \rightarrow \infty}\left[\frac{n}{2} r^2 \sin \left(\frac{2 \pi}{n}\right)\right]=\lim {n \rightarrow \infty}\left[\pi r^2 \frac{\sin \left(\frac{2 \pi}{n}\right)}{\frac{2 \pi}{n}}\right]=\pi r^2 \lim {n \rightarrow \infty}\left[\frac{\sin \left(\frac{2 \pi}{n}\right)}{\frac{2 \pi}{n}}\right]=\pi r^2
$

Questo limite rappresenta l’area del cerchio.

Punto 4.

Si tratta di un famoso problema nato nella matematica greca, ovvero se sia possibile, usando soltanto la riga e il compasso, costruire un quadrato equivalente, cioรจ con la stessa area, di un cerchio dato.

Per il cerchio questo problema non si puรฒ risolvere con riga e compasso.

Questo problema รจ possibile invece per ogni poligono. Ad esempio, รจ possibile fare la “quadratura di un triangolo”; si puรฒ infatti costruire, usando soltanto la riga e il compasso, un quadrato equivalente al triangolo.

Nell’Ottocento รจ stato dimostrato che il problema della quadratura del cerchio รจ impossibile perchรฉ il numero $\pi$ รจ un numero trascendente (ovvero, non รจ soluzione di nessuna equazione algebrica a coefficienti interi).

QUESITO 1

La regione R delimitata dal grafico di $y=2 \sqrt{x}$, dall’asse x e dalla retta $\mathrm{x}=1$ (in figura) รจ la base di un solido S le cui sezioni, ottenute tagliando S con piani perpendicolari all’asse x , sono tutte triangoli equilateri. Si calcoli il volume di S .

Soluzione quesito 1

Il volume del solido si puรฒ ottenere con un integrale definito con il metodo delle “fette”.

Determiniamo la superficie del generico triangolo equilatero. Si ottiene:

$$
A(x)=\frac{1}{2}(2 \sqrt{x}) \cdot\left(2 \sqrt{x} \frac{\sqrt{3}}{2}\right)=x \sqrt{3} .
$$

Integrando si ha:

$$
V=\int_0^1 x \sqrt{3} d x=\frac{\sqrt{3}}{2}\left[x^2\right]_0^1=\frac{\sqrt{3}}{2} .
$$

QUESITO 2

Le misure dei lati di un triangolo sono $40,60 \mathrm{e} 80 \mathrm{~cm}$. Si calcolino, con l’aiuto di una calcolatrice, le ampiezze degli angoli del triangolo approssimandole in gradi e primi sessagesimali.

Soluzione quesito 2

Poichรฉ si conoscono i tre lati del triangolo, allora si deve applicare come primo passaggio il teorema del coseno (o di Carnot).

Si ottiene:

$
a^2=b^2+c^2-2 b c \cos \alpha
$

$$
\cos \alpha=\frac{b^2+c^2-a^2}{2 b c}
$$

Quindi

$
\cos \alpha=\frac{b^2+c^2-a^2}{2 b c}
$

Sostituendo $a=80, b=60, c=40$, si ottiene $\cos \alpha=-\frac{1}{4}$, da cui $\alpha=\arccos \left(-\frac{1}{4}\right)$.

Si ha $\alpha \approx 104^{\circ} 28,7^{\prime}$.

Analogamente si ha:

$
\cos \beta=\frac{a^2+c^2-b^2}{2 a c}
$

$\cos \beta=\frac{11}{16}$, da cui $\beta=\arccos \left(\frac{11}{16}\right) \approx 46^{\circ} 34^{\prime}$

Infine si ha $\cos \gamma=\frac{7}{8}$. Quindi $\gamma=\arccos \left(\frac{7}{8}\right) \approx 28^{\circ} 57,3 \ldots$

QUESITO 3

Si determini, al variare di $k$, il numero delle soluzioni reali dell’equazione:

$$
x^3-x^2-k+1=0
$$

Soluzione quesito 3

L’equazione data puรฒ essere scritta nella forma:

$
x^3-x^2=k-1
$

che si puรฒ risolvere tramite il sistema

$
\begin{cases}
y=x^3-x^2 \\
y=k-1
\end{cases}
$

La cubica $y=x^3-x^2$ รจ facile da studiare e da disegnare.

La derivata prima รจ $y^{\prime}=3 x^2-2 x$.

Il massimo si ha per $x=0$ e il minimo relativo $x=\frac{2}{3}$.

Il minimo relativo vale $f\left(\frac{2}{3}\right)=-\frac{4}{27}$.

Si conclude che l’equazione ammette una soluzione per $k>1$ oppure per $k<\frac{23}{27}$. Per $\frac{23}{27} \leq k \leq 1$, l’equazione ammette tre soluzioni (con due di esse eventualmente coincidenti per i valori estremi, ossia per $k=1$ e per $k=\frac{23}{27}$ ).

QUESITO 4

Un serbatoio di olio ha la stessa capacitร  del massimo cono retto di apotema 1 metro. Si dica quanti litri d’olio il serbatoio puรฒ contenere.

Soluzione quesito 4

Si pone $h=\overline{O V}=x$. Si ricava pertanto il raggio di base del cono $r=\sqrt{1-x^2}$ con $0 \leq x \leq 1$.

Il volume del cono รจ dato da:

$$
V(x)=\frac{1}{3} \pi\left(1-x^2\right) x
$$

Derivando si trova che il volume รจ massimo per $x=\frac{1}{\sqrt{3}}$. In questo caso si ha:

$
V=\frac{1}{3} \pi \frac{2}{3} \frac{1}{\sqrt{3}}=\frac{2 \pi \sqrt{3}}{27} \approx 0,40307 \mathrm{~m}^3
$

Trasformando in litri, si ottiene:

$$
V \approx 403 \text { litri. }
$$

QUESITO 5

Si mostri che la funzione $y=x^3+8$ soddisfa le condizioni del teorema del valor medio (o teorema di Lagrange) sull’intervallo $[-2,2]$. Si determinino i valori medi forniti dal teorema e se ne illustri il significato geometrico.

Soluzione quesito 5

La funzione data รจ ottenuta traslando di 8 verso l’alto la funzione $y=x^3$.

E’ una funzione derivabile e quindi continua in R e quindi anche nell’intervallo dato $[-2 ; 2]$.

In base al teorema di Lagrange citato nell’enunciato esiste almeno un punto $x_0$ interno all’intervallo tale che:

$$
\frac{f(b)-f(a)}{b-a}=f^{\prime}\left(x_0\right)
$$

Per determinare tale punto (o tali punti) dobbiamo risolvere l’equazione

$$
\frac{f(2)-f(-2)}{2-(-2)}=f^{\prime}(x)
$$

ossia

$3 x^2=4$

Si ha pertanto $x= \pm \frac{2}{\sqrt{3}}$.

In questi punti del grafico della cubica la retta tangente รจ parallela alla corda che congiunge i punti estremi A $(-2 ; 0)$ e B $(2 ; 16)$ dell’arco della curva.

QUESITO 6

Si sa che il prezzo $p$ di un abito ha subito una maggiorazione del $6 \% \mathrm{e}$, altresรฌ, una diminuzione del $6 \%$; non si ha ricordo, perรฒ, se sia avvenuta prima l’una o l’altra delle operazioni. Che cosa si puรฒ dire del prezzo finale dell’abito?

Soluzione quesito 6

Il prezzo finale dell’abito รจ inferiore a quello iniziale ed รจ indifferente, in questo caso se si รจ avuto prima l’aumento e poi la riduzione.
Infatti in entrambi i casi, il prezzo finale sarร :

$$
p^{\prime}=p \cdot\left(1-\frac{6}{100}\right) \cdot\left(1+\frac{6}{100}\right)=p \cdot 0,94 \cdot 1,06=p \cdot 0,9964
$$

Il prezzo finale รจ quindi il $99,64 \%$ del prezzo iniziale.
In generale se $t$ รจ il tasso percentuale di riduzione e di aumento, si ha:

$$
p^{\prime}=p \cdot(1-t) \cdot(1+t)=p \cdot\left(1-t^2\right)<p
$$

QUESITO 7

Se $f(x)$ รจ una funzione reale dispari (ossia il suo grafico cartesiano รจ simmetrico rispetto all’origine), definita e integrabile nell’intervallo $[-2,2]$, che dire del suo integrale esteso a tale intervallo? Quanto vale nel medesimo intervallo l’integrale della funzione $3+f(x)$ ?

Soluzione quesito 7

Se la funzione รจ dispari (e continua) allora il suo integrale in un intervallo simmetrico rispetto allโ€™origine O รจ nullo.

L’integrale della funzione $3+f(x)$ sarร  pertanto 12 :

$$
\int_{-2}^2(3+f(x)) d x=\int_{-2}^2 3 d x+\int_{-2}^2 f(x) d x=12
$$

QUESITO 8

Si risolva l’equazione $4\binom{n}{4}=15\binom{n-2}{3}$

Soluzione quesito 8

Il numero $n$ deve essere un numero naturale maggiore od uguale a 5 perchรฉ deve essere $n-2 \geq 3$. Applicando la definizione di coefficiente binomiale:

$$
4 \frac{n(n-1)(n-2)(n-3)}{4 \cdot 3 \cdot 2 \cdot 1}=15 \frac{(n-2)(n-3)(n-4)}{3 \cdot 2 \cdot 1}
$$

Semplificando si ottiene:

$n(n-1)=15(n-4)$

Si ottiene l’equazione di secondo grado:

$n^2-16 n+60=0$

che ha le soluzioni, entrambe accettabili, $n=6$ e $n=10$.

QUESITO 9

Si calcoli l’integrale indefinito $\int \sqrt{1-x^2} d x$ e, successivamente, si verifichi che il risultato di $\int_0^1 \sqrt{1-x^2} d x$ รจ in accordo con il suo significato geometrico.

Soluzione quesito 9

Lโ€™integrale indefinito assegnato si risolve per sostituzione, una sostituzione di tipo trigonometrico; la sostituzione รจ suggerita dalla seguente figura.

Si pone $x=\cos t$ e si ricava $d x=-\sin t \cdot d t$.

Quindi $t=\arccos x$

Sostituendo nell’integrale $\int \sqrt{1-x^2} d x$ si ha

$$
\int \sqrt{1-\cos ^2 t}(-\sin t) d t==-\int \sin ^2 t d t=-\int \frac{1-\cos 2 t}{2} d t=-\frac{t}{2}+\frac{1}{4} \sin 2 t+c
$$

Quindi tornando alla variabile $x$ si ha:

$$
\int \sqrt{1-x^2} d x=-\frac{1}{2} \arccos x+\frac{1}{2} x \sqrt{1-x^2}+c
$$

In definitiva si ha

$$
\int_0^1 \sqrt{1-x^2} d x=\frac{1}{2}\left[-\arccos x+x \sqrt{1-x^2}\right]_0=\frac{1}{2}(\arccos 0)=\frac{\pi}{4}
$$

L’integrale corrisponde a un quarto dell’area del cerchio unitario: $\frac{\pi}{4}=\int_0^1 \sqrt{1-x^2} d x$.

QUESITO 10

Per orientarsi sulla Terra si fa riferimento a meridiani e a paralleli, a latitudini e a longitudini. Supponendo che la Terra sia una sfera $S$ e che l’asse di rotazione terrestre sia una retta $r$ passante per il centro di $S$, come si puรฒ procedere per definire in termini geometrici meridiani e paralleli e introdurre un sistema di coordinate geografiche terrestri?

Soluzione quesito 10

Un meridiano (figura 1) รจ una delle due semicirconferenze massime ottenute dallโ€™intersezione di un piano passante per il polo Nord e per il polo Sud con la superficie sferica della Terra.

Gli estremi di un meridiano sono dunque il polo Nord e il polo Sud (figura 1).

Un parallelo (figura 2) รจ una circonferenza ottenuta dallโ€™intersezione tra un piano perpendicolare allโ€™asse terrestre con la superficie terrestre.

Considerato un meridiano di riferimento (Greenwich), per ogni punto della superficie terrestre (diverso dai poli) passa uno ed un solo meridiano.

Lโ€™angolo (minore di 180ยฐ) formato tra il piano passante per lโ€™asse terrestre contenente il meridiano e quello di riferimento si chiama longitudine (che รจ detta latitudine Est oppure latitudine Ovest a seconda che il punto P si trovi a Est oppure a ovest del meridiano di Greenwich. Ad esempio lโ€™Italia รจ a Est; vedi figura 3).

Considerato il piano perpendicolare allโ€™asse terrestre e passante per il centro O della Terra, questo interseca la superficie sferica su un cerchio massimo detto equatore. Il meridiano passante per P incontra lโ€™equatore in un punto E.

Lโ€™angolo POE si chiama latitudine (โ€œlatitudine Nordโ€ se P appartiene allโ€™emisfero Nord, โ€œlatitudine Sudโ€ se P appartiene allโ€™altro emisfero; vedi figura 4).

SOS Matematica

4.6
SCARICA